Eu tentaria algo do tipo:

  p^n = (q - 12)(q + 12)

Logo, tem-se o sistema:

  p^n1 = q - 12
  p^n2 = q + 12

com n1 e n2 inteiros nao negativos
(no caso, agora n1 OU EXCLUSIVO n2 pode ser nulo)
tais que (n1 + n2) = n.

Do sistema, p^n2 - p^n1 = 24

Assim, a diferenca de duas potencias do primo p
deve ser igual a 24.
Testando para os primos conhecidos
(vou considerar apenas os primos positivos.

p = 2: 2^5 - 2^3 = 32 - 8 = 24
       n2 = 5, n1 = 3 => n = 8 e q = 20
p = 3: 3^3 - 3^1 = 27 - 3 = 24
       n2 = 3, n1 = 1 => n = 4 e q = 15
p = 5: 5^2 - 5^0 = 25 - 1 = 24
       n2 = 2, n1 = 0 => n = 2 e q = 13

Logo, as solucoes sao:
(p,n,q) = (2,8,20), (3,4,15), (5,2,13)




On Tue, 3 Nov 2009 02:48:14 -0800 (PST), luiz silva wrote
> Vamos tentar :
>  
> p^n = q^2 - 12^2
> 
> 1) Para  0<q^2<12^2 temos (testando mesmo) :
>  
> q=+-11, p=23, n=1
> q=+-1, p=143, n=1
>  
> 2) Para n 2 ou 4, vc já fez
>  
> 3) Para q2>12^2 e n>2 temos :
>  
> p^n=(q+12)(q-12)
>  
> q+12 = 0 mod p
> q=-12 mod p
>  
> q-12=0 mod p
>  
> -24=0 mod p
>  
> p=2 ou p=3
>  
> p = 2 teremos : 2^n = q^2 - 12^2
>  
> q=2qo   
>  
> 2^n = 4qo^2 - 12^2
> 2^(n-2)=qo^2 - 6^2
>  
> qo=2q1
>  
> 2^(n-2) = 4q1^2 - 6^2
> 2^(n-4) = q1^2 - 3^2
>  
> mdc (q+3, q-3)= 2
> Como q+3>q-3 temos que q-3 = 2 e q+3 = 2^(n-5)
> Temos então q=5 e 8=2^(n-5) Com isso, n-5=3, n=8.
>  
> Como 2^a deixa resto 1 ou 2 por três (um se a for par, e 2 se a for ímpar) 
> e b^2 deixa resto zero ou um por três, temos que n deverá ser um número 
> ímpar, 
> e qo=1 mod 3    3^n = q^2 - 12^2   3^n = (q+12)(q-12)   mdc(q+12,q-12)= 3   q-
> 12=3 , q+12 = 3^(n-1)   q=15 27 = 3^(n-1) n-1= 3; n=4.   Acho que é isso !!   
> Abs
> Felipe
> --- Em seg, 2/11/09, marcone augusto araújo borges 
> <marconeborge...@hotmail.com> 
escreveu:
> 
> De: marcone augusto araújo borges <marconeborge...@hotmail.com>
> Assunto: [obm-l] Questão 8 da prova do ime
> Para: obm-l@mat.puc-rio.br
> Data: Segunda-feira, 2 de Novembro de 2009, 23:06
> 
> Seja a equação p^n +144=q^2,onde n e q são números inteiros positivos e p é 
> um 
> número primo.Determine os valores possíveis de n,p e q.Para n=2,temos p=5 e 
> q= 
> 13.Para n=4,temos p=3 e q=15.E isso é muito pouco.Como proceder para os 
> outros 
> valores de n?
> 
> Novo Internet Explorer 8: faça tudo com menos cliques. Baixe agora, é gratis!
> 
>       
____________________________________________________________________________________
> Veja quais são os assuntos do momento no Yahoo! +Buscados
> http://br.maisbuscados.yahoo.com
> -- 
> This message has been scanned for viruses and
> dangerous content by MailScanner, and is
> believed to be clean.


Sergio Lima Netto
PEE-COPPE/DEL-Poli/UFRJ
POBox 68504, Rio de Janeiro, RJ
21941-972, BRAZIL
(+55 21) 2562-8164


-- 
This message has been scanned for viruses and
dangerous content by MailScanner, and is
believed to be clean.

=========================================================================
Instru��es para entrar na lista, sair da lista e usar a lista em
http://www.mat.puc-rio.br/~obmlistas/obm-l.html
=========================================================================

Responder a